Find the area of square A B C D ABCD

Geometry Level 4

On square A B C D ABCD , points E , F , G E,F,G , and H H lie on sides A B , B C , C D , \overline{AB},\overline{BC},\overline{CD}, and D A , \overline{DA}, respectively, so that E G F H \overline{EG} \perp \overline{FH} and E G = F H = 34 EG=FH = 34 . Segments E G \overline{EG} and F H \overline{FH} intersect at a point P P , and the areas of the quadrilaterals A E P H , B F P E , C G P F , AEPH, BFPE, CGPF, and D H P G DHPG are in the ratio 269 : 275 : 405 : 411. 269:275:405:411. Find the area of square A B C D ABCD .


The answer is 850.

This section requires Javascript.
You are seeing this because something didn't load right. We suggest you, (a) try refreshing the page, (b) enabling javascript if it is disabled on your browser and, finally, (c) loading the non-javascript version of this page . We're sorry about the hassle.

1 solution

Johanz Piedad
Sep 12, 2015

Notice that 269 + 411 = 275 + 405 269+411=275+405 . This means E G \overline{EG} passes through the centre of the square.

Draw I J H F \overline{IJ} \parallel \overline{HF} with I I on A D \overline{AD} , J J on B C \overline{BC} such that I J \overline{IJ} and E G \overline{EG} intersects at the centre of the square O O .

Let the area of the square be 1360 a 1360a . Then the area of H P O I = 71 a HPOI=71a and the area of F P O J = 65 a FPOJ=65a . This is because H F \overline{HF} is perpendicular to E G \overline{EG} (given in the problem), so I J \overline{IJ} is also perpendicular to E G \overline{EG} . These two orthogonal lines also pass through the center of the square, so they split it into 4 congruent quadrilaterals.

Let the side length of the square be d = 1360 a d=\sqrt{1360a} .

Draw O K H I \overline{OK}\parallel \overline{HI} and intersects H F \overline{HF} at K K . O K = d H F J I A B C D = d 10 OK=d\cdot\frac{HFJI}{ABCD}=\frac{d}{10} .

The area of H K O I = 1 2 H F J I = 68 a HKOI=\frac12\cdot HFJI=68a , so the area of P O K = 3 a POK=3a .

Let P O = h $ . T h e n $ K P = 6 a h \overline{PO}=h\$. Then \$KP=\frac{6a}{h}

Consider the area of P F J O PFJO . 1 2 ( P F + O J ) ( P O ) = 65 a \frac12(PF+OJ)(PO)=65a ( 17 3 a h ) h = 65 a (17-\frac{3a}{h})h=65a h = 4 a h=4a Thus, K P = 1.5 KP=1.5 .

Solving ( 4 a ) 2 + 1. 5 2 = ( d 10 ) 2 = 13.6 a (4a)^2+1.5^2=(\frac{d}{10})^2=13.6a , we get a = 5 8 a=\frac58 .

Therefore, the area of A B C D = 1360 a = 850 ABCD=1360a=\boxed{850}

AIME 2014 Problem

Department 8 - 5 years, 6 months ago

Excellent problem! Is in original?

Dmitry Lebedev - 1 year, 5 months ago

1 pending report

Vote up reports you agree with

×

Problem Loading...

Note Loading...

Set Loading...